How can I solve this problem

How Can I Solve This Problem

Answers

Answer 1

Answer:

x = 27

Step-by-step explanation:

I'm going to use letters to label the parts of the triangle while explaining how to solve for x:

Angle A = x (bottom left angle of the triangle)

Angle B = 4x (top angle of the triangle)

Angle C = empty angle (bottom right angle of the triangle)

Angle D = 3x + 54 (angle outside of the triangle)

A line has a angle measurement of 180 degrees.

Angle C & D make up the line, meaning Angle C + Angle D = 180 degrees.

To find Angle C, we would subtract Angle D's equation from 180 like this:

180 - (3x + 54)

Now Angle C equals 180 - (3x + 54)

Now we have all the angles inside the triangle. To find x, we are going to sum up  Angle A, B, & C and set it equal to 180.

We set the equation to 180 because a the sum of the interior angles for a triangle is 180.

The equation looks like this:

(180 - (3x + 54)) + 4x + x = 180

Now we use basic algebra to solve it:

(180 - (3x + 54)) + 4x + x = 180

(distribute the - to (3x + 54))

180 - 3x - 54 + 4x + x = 180

(add like terms on the left side of the equation)

126 + 2x = 180

(subtract 126 from both sides)

2x = 54

(divide both sides by 2)

x = 27

To prove this answer is correct, plug it back in the original equation and you'll see that it ends up equaling 180, which is the sum of interior angles for triangle.

Hope it helps (●'◡'●)


Related Questions

Write the equation of the graph y=

Answers

9514 1404 393

Answer:

  y = 6^x -3

Step-by-step explanation:

The graph is that of an exponential function that has been translated downward. We notice the horizontal asymptote is -3, and a couple of points on the graph are (0, -2) and (1, 3).

The shifted parent function will look like ...

  y = a·b^x +c

where c is the horizontal asymptote. Using the two points we found, we have ...

  -2 = a·b^0 -3 . . . . . using (x, y) = (0, -2)

  1 = a . . . . . . . . . . add 3 and simplify

Then using (x, y) = (1, 3), we have ...

  3 = b^1 -3

  6 = b . . . . . . . . . add 3 and simplify

So, the equation is ...

  y = 6^x -3

what is the value of b?

Answers

Answer: The b-value is the middle number, the number next to the X

if a real estate company gives 3% commission on to it's agents. if an agent sold a piece of land for 12,50,000, how much cccommision did the agent get ? ​

Answers

Answer:

37500

Step-by-step explanation:

3% of 1250000

That is (3/100)*1250000

The answer is 37500

Please help :(((((((((((((((((((((((((

Answers

jeez that looks really hard

Step-by-step explanation:

BRAINLIEST FOR ANSWER WITH EXPLANATION
JUST ONE QUESTION ATTACHED BELOW:

Answers

Answer:

m=16

Step-by-step explanation:

(6+6)/m=6/8

12/m=6/8

M=16

Answer:

16

Step-by-step explanation:

This triangle has been dialated; ILM is a dialated version of IJK. We know that dialations are by scale factors, and we also know the length of two corresponding sides.

IM is 6 m long, and IMK is 6+6 m long, so 12 m. So using simple ratios, we know the dialation factor from ILM to IJK is 2. The corresponding side for JK (m) is LM, which we know is 8 m. So side m is 16 because of the scale factor (2).

Maybe you thought we were solving for "m", as in a varriable all the sides are multiplied by, but I think that's just bad labeling. Hope this helps!

a train leaves Westchester at 6:30. What time should it arrive at Middlewich

Answers

Answer:

a). 6:51

b). 6:30 am

c). 34 minutes

Step-by-step explanation:

a). Train leaves Westchester at 6:30.

   From the arrival - departure table in column (2),

   Arrival time of the train at middlewich = 6:51

b). Kate has to reach Southam before 9:00 am

   Therefore, time of the latest train that she can catch to get to work on time is 6:30 am

    By this train she can reach at 07:19 at Southam.

c). Duration of journey from Westchester to Eastwick = 06:34 - 06:00

                                                                                        = 00:34

                                                                                        ≈ 34 minutes

Find the measure of the indicated angle

Answers

Answer:

x = 70°

Step-by-step explanation:

Given is the isosceles triangle as two sides marked equal.

The missing angles measure the same.

Since the sum of interior angles of a triangle is 180°, we have:

40° + 2x = 180°2x = 140°x = 70°

It is an isosceles triangle,

The marked 2 sides of triangle are equal.

Now we have to,

find the measure of indicated angle.

General formula,

→ Sum of all sides of triangle = 180°

Then the value of x is,

→ 2x + 40° = 180°

→ 2x = 140°

→ x = 140/2

→ x = 70°

Therefore, 70° is the value of x.

Find the equation of the line through point (2,2) and parallel to y=x+4. Use a forward slash (i.e.”/“) for fractions (e.g. 1/2 for

Answers

Answer:

The equation of the line is, y = x

Step-by-step explanation:

The constraints of the required linear equation are;

The point through which the line passes = (2, 2)

The line to which the required line is parallel = y = x + 4

Two lines are parallel if they have the same slope, therefore, we have;

The slope of the line, y = x + 4 is m = 1

Therefore, the slope of the required line = 1

The equation of the required lime in point and slope form becomes;

y - 2 = 1 × (x - 2)

∴ y = x - 2 + 2 = x

The equation of the required line is therefore, y = x

Here is a Venn diagram.
a) Write down the numbers that are in set
А.
B
16
(i) AUB 8,14, 17, 16, 9, 15
8
14
9
(ii) A B 14,17
17
15
13
12
10
11
One of the numbers in the diagram is
chosen at random.
b) Find the probability that the number is in
set A, giving your answer as a decimal.

Answers

Answer:

0.7

Step-by-step explanation:

.

Answer:  b) 0.7

Just copy that into mathswatch :>

Does this graph show a function? Explain how you know.
• A. No; there are yvalues that have more than one x-value.
O B. No; the graph fails the vertical line test.
O C. Yes; there are no yvalues that have more than one x-value.
O D. Yes; the graph passes the vertical line test.


HELP PLSS

Answers

The answer is d. If you draw a line straight up it doesn’t cross the line more than once.

helpppppppppppppppppp

Answers

Answer:

Dude

sheesh i only seeing 5 and 18
The answer is 5 it is the only number that can be multiplied by another number to get 10

One number is 6 times a second number. The sum of the two numbers is 56. Find the numbersOne number is 6 times a second number.

The sum of the two numbers is 56. Find the numbers

Answers

Answer:

I think its 56 = 56

Step-by-step explanation:

Answer:

8 and 48

Step-by-step explanation:

This can be written as a system of equations. X will be the smaller number and y will be the larger:

6x = y

x + y = 56

Then substitute y in the second equation for 6x(seen in first equation) and solve.

x + 6x = 56

7x = 56

x = 8

So one number is 8. To find the other plug in 8 for x in either of the original equations (both get the same answer for y).

6x = y

6 * 8 = y

48 = y

Then double check by seeing if they fit the requirements of the problem.

Hope this helps!

Word problem One of the citizens has 97 silver coins. How many bronze coins would it take to equal this amount

Answers

Given: Given that a citizen have 97 silver coins.

To find : Here we need to find that how many bronze coins would it take to equal this amount.

Solution: We know, 1 silver coin=10 bronze coin

So, 97 silver coin=10×97 bronze coin

=970 bronze coin

Therefore, 970 bronze coins would it take to equal this amount.

Surface area of a cuboid is 384
work out the volume

Answers

Answer:

is 512

Step-by-step explanation:

Someone tell me where everyone is going right please !!

Answers

Answer:

1min = 0.25miles

5.25miles / 0.25miles = 25 = 25minutes

Step-by-step explanation:

Hope this is right I'm not the best at worded time/distance math questions.

9514 1404 393

Answer:

  0 ≤ t < 52.5 minutes

Step-by-step explanation:

Riko will be behind Yuto until her distance traveled matches his. That is, she will be behind for ...

  0.35t < 5.25 +0.25t

  0.10t < 5.25

  t < 52.5

Riko will be behind Yuto on the interval 0 ≤ t < 52.5 minutes.

_____

Additional comment

distance = speed × time

Here, time is measured from when Riko starts riding. In addition to the 5.25 miles that Yuto has already gone, his distance will be the product of his speed (0.25 mi/min) and the travel time (t min). Then Yuto's total distance is 5.25+0.25t miles. The speed×time product is also used to find Riko's distance traveled. In her case, it is 0.35t miles.

someone please help me ASAP!

Answers

Answer:

214°

Step-by-step explanation:

The measure of an arc that sees the center angle of the circle is equal to the very same angle that it sees

Since the measure of circle is 360° and arc AB is given as 146° the measure of arc ACB should be 360 - 146 = 214°

Two particles have positions at time t given by s1=4t-t^2 and s2=5t^2-t^3. Find the velocities

Answers

Step-by-step explanation:

Given that,

The positions of the first particle,[tex]s_1=4t-t^2[/tex]

Velocity,

[tex]v_1=\dfrac{ds_1}{dt}\\\\v_1=\dfrac{d(4t-t^2)}{dt}\\\\v_1=4-2t[/tex]

Position,

[tex]s_2=5t^2-t^3[/tex]

[tex]v_2=\dfrac{ds_2}{dt}\\\\v_2=\dfrac{d(5t^2-t^3)}{dt}\\\\v_2=10t-3t^2[/tex]

Hence, this is the required solution.

PLEASE ANSWER ASAPPP

Answers

Answer:

the answer is 2035.75 cm³

Step-by-step explanation:

comment if you want explanation

when x^2 - 3x +2k is divided by x+2, the remainder is 7. find the value of k.

Answers

Answer:

let's find the zeros of the divisor

x+2=0

x=-2

let x²-3x+2k =f(x)

Now f(-2)=(-2)²-3(-2)+2k=7

4+6+2k=7

2k=7-10

2k=-3

k=-3/2

The number of clicks for a search text ad is 50 and the number of impressions is 5000. The CTR would be Group of answer choices 1% 2% 5% 10%

Answers

Given:

Clicks = 50

Impressions = 5000

To find:

The CTR percentage.

Solution:

We know that,

[tex]CTR=\dfrac{\text{Clicks}}{\text{Impressions}}\times 100[/tex]

Substituting the given values, we get

[tex]CTR=\dfrac{50}{5000}\times 100[/tex]

[tex]CTR=\dfrac{1}{100}\times 100[/tex]

[tex]CTR=1\%[/tex]

Therefore, the correct option is A.

which points lie in the second quadrant? check all that apply.

Answers

Point F

Point E

These lie in the second quadrant.

Can someone help me with this math homework please!

Answers

4. i believe would be D. subtract 1.1j from both sides.
5. x <_ 3
6. y=10

Answer:

Question 1: (C) Step 3

Question 2: (B) The 2nd graph from the left

Question 3: (A, B, D)

Question 4: (D) Subtract 1.1j from both sides

Question 5: (C) x <= 3

Question 6: (D) y = 10

Step-by-step explanation:

Question 1:

Step 3 is where Rahul added 1/4 to both sides. This is the only time he used the addition property of equality.

Question 2:

This questions requires you to have knowledge on how to graph lines.

g(x) = -1 is already graphed correctly on all of the answer choices, so we only have to focus on f(x) = -1/2x - 2.

The y-intercept for f(x) is -2, and only answers B & D have that intercept. However, only B has the correct slope of -1/2.

Question 3:

A is correct because they added 3/5x and x, which is still equal to the original equation.

B is correct because they multiplied all the terms to have a denominator of 30, then multiplied both sides by 30 to eliminate the fractions.

C is wrong because it's supposed to 30x, not just a regular x from the original equation.

D is correct because as you solve the equation for x, you end up adding 18x, 6x, and 30x, which is 54x. 24x + 30x in this answer choice also equals 54x.

E is wrong because they subtracted instead of adding the 6x.

Question 4:

All of the answer choices are wrong except for D because the other choices either use a wrong term (like 4j, which doesn't exist) or the wrong property of equality.

Question 5:

Use basic algebra to sovle the inequality:

2(4 + 2x) >= 5x + 5

(distribute 2 to the (4 + 2x))

8 + 4x >= 5x + 5

(subtract 5 from both sides)

3 + 4x >= 5x

(subtract 4x form both sides)

3 >= x

x <= 3

Question 6:

Use basic algebra to solve for y:

2.8y + 6 + 0.2y = 5y - 14

(subtract 6 from both sides)

2.8y + 0.2y = 5y -20

(add like terms)

3y = 5y - 20

(subtract 5y from both sides)

-2y = -20

(divide both sides by -2)

y = 10

Hope it helps (●'◡'●)

Hey Guys! Can You Please Help Me In My Math​

Answers

This would be a great tree diagram
Sorry...

pic is not clear.....

Answer thi question plz

Answers

Answer:

Q.5

Step-by-step explanation:

(2x)^3+3×(2x)^2×3y+3×2x×(3y)^2+(3y)^3

8x^3+3×4x^2 × 3y + 3×2x×9y^2 + 27y^3

8x^3 + 36x^2 × y + 54xy^2 + 27y^3

30 points if helped, question in the picture

Answers

Step-by-step explanation:

[tex] \frac{2}{2 + \sqrt{3} } \times \frac{2 - \sqrt{3} }{2 - \sqrt{3} } = \frac{2(2 - \sqrt{3} )}{1} = 4 - 2 \sqrt{3} [/tex]

Does the point (0, 37) satisfy the equation y = 79x - -37? yes no​

Answers

Answer:

yes

Step-by-step explanation:

y = 79x - -37

37 = 79(0) - - 37

37 = 0 - - 37

37 = 37

Heya Kitties!
What is the value of x?

18x−16=−12x−4



x =

Answers

Answer:

2/5

Step-by-step explanation:

18x+12x=-4+16

30x=12

x=2/5

Answer:

18x-16=-12x-4

18x+12x=-4+16

30x=12

x=12/30

x=2/5

what is the product of -8(9)
show your work to the problem ​

Answers

-72 just do 8*9 and add a negative sign

Answer:

-72

Step-by-step explanation:

-8×-9

=-72

hope it helps you..

please help!
factor f(x)=6x^2-x-2​

Answers

Hi there!

[tex]\large\boxed{(3x - 2)(2x + 1) }[/tex]

6x² - x - 2

To solve, we must factor into the following format:

(ax - b)(cx - d)

The following conditions must be met:

a · c = 6

da + bd = -1

b · d = -2

By guessing and checking, we get:

(3x - 2)(2x + 1)

Answer for Acellus:

(2 x + 1) (3 x - 2)

Step-by-step explanation:

i checked the equation on an online calculator, and it came up with it switched (compared to the other answer that is posted) like so...

I put the answer in and it said it was right for Acellus.

Hope this helps :))

If A = [tex]\left[\begin{array}{ccc}cosx&-sinx\\sinx&cosx\end{array}\right][/tex], then show that [tex](A^{-1} )^{-1}[/tex]

Answers

Given:

The matrix is:

[tex]A=\begin{bmatrix}\cos x&-\sin x\\\sin x&\cos x\end{bmatrix}[/tex]

To show:

[tex](A^{-1})^{-1}=A[/tex]

Solution:

If a matrix is:

[tex]M=\begin{bmatrix}a&b\\c&d\end{bmatrix}[/tex]

Then,

[tex]M^{-1}=\dfrac{1}{ad-bc}\begin{bmatrix}d&-b\\-c&a\end{bmatrix}[/tex]

We have,

[tex]A=\begin{bmatrix}\cos x&-\sin x\\\sin x&\cos x\end{bmatrix}[/tex]

Using the above formula, we get

[tex]A^{-1}=\dfrac{1}{(\cos x)(\cos x)-(-\sin x)(\sin x)}\begin{bmatrix}\cos x&\sin x\\-\sin x&\cos x\end{bmatrix}[/tex]

[tex]A^{-1}=\dfrac{1}{\cos^2x+\sin^2x}\begin{bmatrix}\cos x&\sin x\\-\sin x&\cos x\end{bmatrix}[/tex]

[tex]A^{-1}=\dfrac{1}{1}\begin{bmatrix}\cos x&\sin x\\-\sin x&\cos x\end{bmatrix}[/tex]

[tex]A^{-1}=\begin{bmatrix}\cos x&\sin x\\-\sin x&\cos x\end{bmatrix}[/tex]

Now, the inverse of [tex]A^{-1}[/tex] is:

[tex](A^{-1})^{-1}=\dfrac{1}{(\cos x)(\cos x)-(\sin x)(-\sin x)}\begin{bmatrix}\cos x&-\sin x\\\sin x&\cos x\end{bmatrix}[/tex]

[tex](A^{-1})^{-1}=\dfrac{1}{\cos^2x+\sin^2x}\begin{bmatrix}\cos x&-\sin x\\\sin x&\cos x\end{bmatrix}[/tex]

[tex](A^{-1})^{-1}=\dfrac{1}{1}A[/tex]

[tex](A^{-1})^{-1}=A[/tex]

Hence proved.

Other Questions
Find a power series representation for the function. (Give your power series representation centered at x = 0.) f(x) = x2 x 4 + 81 f(x) = [infinity] n = 0. Can someone help with 16 and 18 kxndjdkdkdkkdkskskdkdjdjdjskskskdjdjddjd How many liters of hydrogen can be produced at a pressure of 2 atm and a temperature of 298 K what is one way mining companies could help protect the environment as part of their business When you did the Homeostasis Exercise Lab yesterday, what was the dependent variable? A) Heart rateB) Breathing rateC) Both Heart rate and Breathing rateD) Time 15 greater than y is at least 150 interval notation The Boston Tea Party: A. was organized by George Washington B. featured mass murder and mayhem C. was an attempt to reconcile the colonists and the British D. led to Britains imposition of the Coercive (Intolerable) Acts While many businesses had to beconverted for the war effort duringWWII, each still managed which of thefollowing?A. a profitB. a deficitC. wage decreases which highlighted word is an adverb? Roll out the ple dough evenly Please Help..!! GERMAN LANGUAGE Please ignore the writing in blue as I tried to work it out but couldnt What is the most important criteria for using chromatography to separate components from a mixture A absoro de gua e nutrientes do solo e fixao de vegetal so funes bsicas: *10 pontosa. Do caule.b. Das folhas.c. Das razes.d. Das flores. How does sustainable development ensure harmony between nature and population? Give your opinion. 21. Who will be at fault if you hit a person getting out of a parked car? A. Nobody, because it is hard to see people getting out of cars. B. The person who wasn't careful and didn't see your car coming. C. You, which is why you should drive slowly and keep a lookout for doors opening or people getting out of cars. D. The city authorities for making city streets so difficult to drive. Gotham City acquires $25,000 of inventory on November 1, 20X7, having held no inventory previously. On December 31, 20X7, the end of Gotham City's fiscal year, a physical count shows $8,000 still in stock. During 20X8, $6,500 of this inventory is used, resulting in a $1,500 remaining balance of supplies on December 31, 20X8.Based on the preceding information, what would be the correct account balances for 20X7 if Gotham City used the purchase method of accounting for inventories? What rules applies to long quotations in MLA in-text citations a) Surround with quotation marks; double space lines b) Indent and start a new line; single space lines c)indent and start a new line; double space lines what number when multiplied by 5 is one third of the sum of 64 and 56? What is the area, in square inches, of the trapezoid below